LSAT and Law School Admissions Forum

Get expert LSAT preparation and law school admissions advice from PowerScore Test Preparation.

 LSAT2018
  • Posts: 242
  • Joined: Jan 10, 2018
|
#49145
I chose answer (A) for this question because the enhancement of athletic performance would suggest that the use of nonaddictive drugs by athletes should be prohibited, thus weakening the argument. I immediately thought of the answer as a possible alternate cause, and I think I didn't look at the remaining answers in more detail. So I would like to ask how answer (A) compares to answer (E), which does seem like another alternate cause. So what makes answer (E) the correct answer?
User avatar
 Dave Killoran
PowerScore Staff
  • PowerScore Staff
  • Posts: 5853
  • Joined: Mar 25, 2011
|
#49193
Hi LSAT,

This is a good example of why you can never assume you have the right answer until you've read every answer completely :-D

Answer choice (E) is correct because if taking these doses of aspirin and vitamins is harmful, then it creates a conflict based on the first sentence of the stimulus, which stated that "Because addictive drugs are physically harmful, their use by athletes is never justified" (italics added). the author can't hold that belief, and then say these drugs—which via (E) are shown to be harmful—should not be prohibited.

With (A), why is this a problem for the author? If the author was presented with this answer, they would simply reply that sure, they enhance performance, but then again so do a lot of other things like running shoes and boxing gloves. As long as they aren't physically harmful, then they are okay.

The difference above is you can see that physical harm is playing a driving role in the author's conclusion.It's why addictive drugs should be banned, and the lack of harm is why nonaddictive drugs are acceptable. but (E) shows they are harmful, which hurts the argument.

Thanks!
 HowardQ
  • Posts: 32
  • Joined: Jun 25, 2018
|
#49413
HI,

I chose answer E simply because none of the other answers is more appealing. However, I am still quite doubtful that "massive doses" was not considered. The conclusion stated use of nonaddictive drugs should be prohibited, not massive doses should be prohibited. The position associated with massive intake belongs to purists, which only associates with them being unnatural. I see this is a weaken question doesn't have to be proven true, but the answer E could be argued to have no effect if lower doses are taken.

Thanks,
 Adam Tyson
PowerScore Staff
  • PowerScore Staff
  • Posts: 5153
  • Joined: Apr 14, 2011
|
#49703
You're right that answer E doesn't prove the claim in the last sentence of the stimulus is false, HowardQ, but it definitely does weaken the argument, because it at least raises some question about whether there should be some prohibition. Purists want to prohibit taking massive doses of those nonaddicitve drugs. The author responds that they should not be prohibited at all ("This is ridiculous"). Since he is attacking the argument made by the purists, anything that strengthens their argument would hurt the authors. Sure, there could be a middle ground where massive doses are prohibited but low doses are not, but the author wants no prohibition at all and answer E gives us reason for at least a partial prohibition.

Get the most out of your LSAT Prep Plus subscription.

Analyze and track your performance with our Testing and Analytics Package.